Efficient Integration of Step Function with Variable Denominator

Click For Summary
To calculate the integral of the step function ## \int^6_{-6} \frac{g(x)}{2+g(x)} \ dx ##, the function can be rewritten using long division. The integral can be simplified to ## 12 - 2\int^6_{-6} \frac{1}{g(x)+2} \ dx ##, where the first part evaluates to 12. The next step involves breaking the integral into segments based on the constant values of g(x) across different intervals. Each segment can be evaluated separately, allowing for a straightforward calculation of the overall integral. This method effectively addresses the challenge of integrating a function with a variable denominator.
Rectifier
Gold Member
Messages
313
Reaction score
4
The problem
I want to calculate ## \int^6_{-6} \frac{g(x)}{2+g(x)} \ dx ## for the step function below.
2YSK7nM.jpg
The attempt
I started with rewriting the function as with the help of long-division
## \int^6_{-6} \frac{g(x)}{2+g(x)} \ dx = \int^6_{-6} 1 \ dx - 2\int^6_{-6} \frac{1}{g(x)+2} \ dx##

I know that ##\int^6_{-6} 1 \ dx = 12## but that's about it. I am not sure how I should continue.

And here is where I get stuck.
 
Last edited:
Physics news on Phys.org
If you look closely at ## g(x) ##, it takes on constant values for various intervals. You need to break up the integral from -6 to 6 into these various segments.
 
  • Like
Likes Rectifier
Rectifier said:
The problem
I want to calculate ## \int^6_{-6} \frac{g(x)}{2+g(x)} \ dx ## for the step function below.
2YSK7nM.jpg
The attempt
I started with rewriting the function as with the help of long-division
## \int^6_{-6} \frac{g(x)}{2+g(x)} \ dx = \int^6_{-6} 1 \ dx - 2\int^6_{-6} \frac{1}{g(x)+2} \ dx##

I know that ##\int^6_{-6} 1 \ dx = 12## but that's about it. I am not sure how I should continue.

And here is where I get stuck.
This shouldn't be too difficult. On the interval [-6, -4], g(x) = -1, so g(x) + 2 = 1. What is ##\int_{-6}^{-4} \frac 1 1 dx##? Do the same for the other intervals.
 
  • Like
Likes Rectifier and Charles Link
Hi Rectifier:

I suggest breaking the integral into six pieces, one piece for each step. For each piece, g(x) has a specific constant value, so the integrand is a specific constant.

Hope this helps.

Regards,
Buzz
 
  • Like
Likes Rectifier
Thank you for your help, everyone!
 
  • Like
Likes Charles Link
Question: A clock's minute hand has length 4 and its hour hand has length 3. What is the distance between the tips at the moment when it is increasing most rapidly?(Putnam Exam Question) Answer: Making assumption that both the hands moves at constant angular velocities, the answer is ## \sqrt{7} .## But don't you think this assumption is somewhat doubtful and wrong?

Similar threads

Replies
3
Views
2K
  • · Replies 4 ·
Replies
4
Views
2K
Replies
9
Views
1K
  • · Replies 105 ·
4
Replies
105
Views
6K
Replies
2
Views
2K
  • · Replies 6 ·
Replies
6
Views
2K
  • · Replies 22 ·
Replies
22
Views
3K
  • · Replies 9 ·
Replies
9
Views
2K
  • · Replies 14 ·
Replies
14
Views
3K
  • · Replies 5 ·
Replies
5
Views
3K